Parameterizing and finding its boundaries (vector calculus)

Click For Summary
The discussion focuses on parameterizing a surface defined by the equation x² + y² + z² = 4, with the constraint z ≥ √2. The parameterization uses spherical coordinates, where x, y, and z are expressed in terms of r, φ, and θ. The confusion arises regarding the boundaries for φ, particularly since z must be between √2 and 2, leading to the condition that φ must be greater than or equal to π/4. It is clarified that the inequality for φ reverses when applying the arc cosine function, as cos(φ) decreases from 1 to 0 in the interval from 0 to π/2. Understanding this reversal is crucial for correctly determining the boundaries for φ in the parameterization.
pcjang
Messages
3
Reaction score
0

Homework Statement



I was studying for my finals, and then the only thing that I got stuck on was parameterizing a surface and finding the area of the surface.

and my problem states that x2 + y2 + z2 = 4
and z \geq\sqrt{2}

Homework Equations



so when parameterizing a sphere, it comes out to be
x = r sin\phicos\theta
y = r sin\phisin\theta
z = r cos\phi



The Attempt at a Solution



so I'm sure that \theta goes from 0 to 2\pi. But i was getting confused what boundaries should be for \phi.

What i tried is that
since z \geq\sqrt{2} and should be less than 2 since 2 is the radius, i put \sqrt{2} \leq z \leq 2.
which z = 2 cos\phi
so when i solved it, it came out to be \pi/4 \leq \phi leq 0.

but it just doesn't make sense to me how the boundary can go backwards. can someone explain to me~??
 
Physics news on Phys.org
oops, last one, i meant to say

phi is greater than or equal to pi/4
and
phi is less than or equal to 0
 
When you have \cos \phi > \frac{1}{\sqrt{2}}, you should note that the graph of cos x decreases continuously from 1 to 0 in the interval 0 to pi/2. Which is why you have to reverse the inequality when taking the arc cos the interval for phi.
 
Question: A clock's minute hand has length 4 and its hour hand has length 3. What is the distance between the tips at the moment when it is increasing most rapidly?(Putnam Exam Question) Answer: Making assumption that both the hands moves at constant angular velocities, the answer is ## \sqrt{7} .## But don't you think this assumption is somewhat doubtful and wrong?

Similar threads

  • · Replies 9 ·
Replies
9
Views
1K
  • · Replies 2 ·
Replies
2
Views
2K
  • · Replies 6 ·
Replies
6
Views
2K
Replies
3
Views
2K
  • · Replies 4 ·
Replies
4
Views
2K
  • · Replies 3 ·
Replies
3
Views
2K
Replies
4
Views
2K
  • · Replies 1 ·
Replies
1
Views
2K
  • · Replies 14 ·
Replies
14
Views
2K